LSAT and Law School Admissions Forum

Get expert LSAT preparation and law school admissions advice from PowerScore Test Preparation.

User avatar
 Dave Killoran
PowerScore Staff
  • PowerScore Staff
  • Posts: 5852
  • Joined: Mar 25, 2011
|
#80540
This game is also discussed in our Podcast: LSAT Podcast Episode 70: The May 2020 LSAT-Flex Logic Games Section

Complete Question Explanation
(The complete setup for this game can be found here: lsat/viewtopic.php?t=33051)

The correct answer choice is (C).

This is an unusual List question since it presents groups of three cities, and asks you to find the answer that won't work for either semester. The easiest wrong answer to spot would be one that violated the third rule and did not contain either M or V. However, no answer fits the bill and thus that quick, backdoor solution fails (no harm in thinking along those lines though!).

The second line of attack—if you see it—would be to realize that a solution that met the sufficient conditions of each of the first two rules (and thus contained both H and V) but then didn't contain the necessary condition of either (so, no M or T) would be destined to fail. While that ultimately produces the correct answer, this is a very high level deduction to make on the fly. So, we'll continue on as if this solution didn't occur to you, but at some point stop and consider why this approach would create issues here because it's the kind of thing they do all the time.

The third line of attack would be to quickly glance at prior work and see if any of the answers match a group that appeared in a viable solution you previously identified. This eliminates several answers:

  • The solution to #12 shows H, M, and then either O or T as a viable group in the spring; this eliminates answers choice (A).

    The hypothetical solution used in #14 (A) and (B) shows H, T, and V as a viable group in the spring; this eliminates answers choice (D).

    The solution to #14 (E) shows H, M, and V as a viable group in the fall; this eliminates answers choice (B).
At this juncture I would likely move to complete the remaining questions in the game in the hopes that one of them would kill off the remaining wrong answer choice. Regrettably, that doesn't happen, but again, there's no harm in thinking along those lines though—that type of aggression typically pays off over the long haul!

Answer choice (A): As discussed above, this answer choice can occur as was shown in question #12.

Answer choice (B): As discussed above, this answer choice can occur as was shown in question #14.

Answer choice (C): This is the correct answer, and this is the second-line-of-attack solution discussed briefly above in the abstract.

This solution could only work in the spring, since placing H in the fall would trigger the first rule, and there is not room for M in the fall. But, placing H, O, and V in the spring then triggers the second rule, but there is now no room for T in the spring. Thus, regardless of where this group of three is placed, one of the rules will be violated. Since there can be no viable solution, this answer choice cannot occur and is correct.

Essentially, this solution meets the sufficient condition of each of the first two rules (and thus contains both H and V) but then doesn't contain the necessary condition of either (so, no M or T). Thus, there won't be enough room for that necessary condition city when the sufficient gets triggered (with H in fall, or with V in the spring), leading to problems.

Answer choice (D): As discussed above, this answer choice can occur as was shown in question #14.

Answer choice (E): This solution could only work in the fall, since placing V in the spring would trigger the second rule, and there is not room for T in the spring. The following solution shows how this answer is possible:

  • ..... ..... _V_ ..... ..... ..... _H_

    ..... ..... _O_ ..... ..... ..... _T_

    ..... ..... _M_ ..... ..... ..... _M/V_
    ..... ..... Fall ..... ..... ..... Spring
 mollylynch
  • Posts: 62
  • Joined: Jul 21, 2023
|
#103315
For this question, V out of HOV is not allowed to have both F and S? Only allowed to have one season?
B/c if not I have...
H: F
M: S
O: F
T: S
V: F/S

And I think this follows the rules
User avatar
 Dave Killoran
PowerScore Staff
  • PowerScore Staff
  • Posts: 5852
  • Joined: Mar 25, 2011
|
#103320
Hi Molly,

The scenario you propose violates the first rule, because you have H in the fall but then M is not in the fall but in the spring.

Thanks!
 mollylynch
  • Posts: 62
  • Joined: Jul 21, 2023
|
#103590
Thank you!

Get the most out of your LSAT Prep Plus subscription.

Analyze and track your performance with our Testing and Analytics Package.